Simple Vector Calc: Homework Solutions

  • Thread starter Thread starter icystrike
  • Start date Start date
  • Tags Tags
    Vector
Click For Summary
The discussion centers on a poorly stated homework problem involving parametric equations for a motorcycle's path. The lack of clarity regarding the time variable t is highlighted as a significant issue, making the problem unsolvable without assumptions. Participants express concerns about the implications of this negligence on the correctness of their solutions. There is also a brief debate over the correctness of a specific calculation related to the problem. Overall, clarity in problem statements is emphasized as crucial for accurate solutions.
icystrike
Messages
444
Reaction score
1

Homework Statement



attachment.php?attachmentid=33722&stc=1&d=1301507870.jpg


Homework Equations





The Attempt at a Solution



attachment.php?attachmentid=33723&stc=1&d=1301507870.jpg
 

Attachments

  • qn.jpg
    qn.jpg
    20.1 KB · Views: 516
  • sol.jpg
    sol.jpg
    10.7 KB · Views: 392
Physics news on Phys.org
My only comment would be that the problem is very poorly stated. It is one thing to say the path is given parametrically by r(t) = <t, (1/2)t2> and another thing to say that r(t) describes the position of the motorcycle at time t, t in seconds, which it does not state. The problem is not solvable without assuming that.
 
oh... i think its their negligence.. But if they have stated that.. will my working be correct? :) Thanks again!
 
icystrike said:
oh... i think its their negligence.. But if they have stated that.. will my working be correct? :) Thanks again!

I think mostly correct. I'm disagreeing with your N by a factor of 1/sqrt(2), but I might be off so check it.
 
Question: A clock's minute hand has length 4 and its hour hand has length 3. What is the distance between the tips at the moment when it is increasing most rapidly?(Putnam Exam Question) Answer: Making assumption that both the hands moves at constant angular velocities, the answer is ## \sqrt{7} .## But don't you think this assumption is somewhat doubtful and wrong?

Similar threads

  • · Replies 4 ·
Replies
4
Views
2K
  • · Replies 4 ·
Replies
4
Views
1K
  • · Replies 2 ·
Replies
2
Views
3K
  • · Replies 2 ·
Replies
2
Views
2K
  • · Replies 4 ·
Replies
4
Views
2K
  • · Replies 3 ·
Replies
3
Views
3K
  • · Replies 4 ·
Replies
4
Views
2K
  • · Replies 14 ·
Replies
14
Views
2K
  • · Replies 16 ·
Replies
16
Views
2K
  • · Replies 3 ·
Replies
3
Views
3K